K
Khách

Hãy nhập câu hỏi của bạn vào đây, nếu là tài khoản VIP, bạn sẽ được ưu tiên trả lời.

6 tháng 9 2016

Ta có M = \(\left(5+2\sqrt{6}\right)^{1004}+\left(5-2\sqrt{6}\right)^{1004}\)

Ta có a2 = 10a - 1 ; b2 = 10b  -1 

Đặt Sn = an + bn 

=> \(a^{n+2}+b^{b+2}=10.\left(a^{n+1}+b^{n+1}\right)-\left(a^n+b^n\right)\)

\(=>s_{n+2}=s_{n+1}.10+s_n\)chia hết cho 10

=> \(s_n+s_{n+2}\)chia hết cho 10

Tương tự ta được \(s_{n+2}+s_{n+4}\)chia hết cho 10

=> \(s_{n+2}+s_{n+4}-s_n-s_{n+2}\)chia hết cho 10

=> \(s_{n+4}-s_n\)chia hết cho 10

Ta có S0 = 2

S1 = 10

=> s2;s3....sn là các số tự nhiên và s0;s4;...;s4n có chữ số tận cùng là 2 

Vậy M có chữ số tận cùng là 2 

6 tháng 6 2017

Bài 3:

Tự CM: 1.2007<2.2006<...<1004.1004(cái này lớp 5 nhé)

SUy ra \(\sqrt{1.2007}< \sqrt{2.2006}< ...< \sqrt{1004.1004}=1004\)

Có: \(S=2\left(\sqrt{1.2007}+\sqrt{3.2005}+...+\sqrt{1003.1005}\right)\)

\(S< 2\left(\sqrt{1004.1004}+\sqrt{1004.1004}+...+\sqrt{1004.1004}\right)\)

\(S< 2.\left(1004+1004+...+1004\right)=2.502.1004=1004.1004=1004^2\)

Suy ra đpcm. BẤM ĐÚNG CHO T NHÉ

6 tháng 10 2017

a/ Ta chứng minh: \(B=\left(\sqrt{3}+\sqrt{2}\right)^{2n}+\left(\sqrt{3}-\sqrt{2}\right)^{2n}=\left(5+2\sqrt{6}\right)^n+\left(5-2\sqrt{6}\right)^n\) là số nguyên với mọi n

Với \(n=0\Rightarrow B=2\)

Với \(n=1\Rightarrow B=10\)

Giả sử nó đúng đến \(n=k\) hay

\(\hept{\begin{cases}\left(5+2\sqrt{6}\right)^{k-1}+\left(5-2\sqrt{6}\right)^{k-1}=a\\\left(5+2\sqrt{6}\right)^k+\left(5-2\sqrt{6}\right)^k=b\end{cases}}\) \(\left(a,b\in Z\right)\)

Ta chứng minh nó đúng đến \(n=k+1\)

Ta có: \(\left(5+2\sqrt{6}\right)^{k+1}+\left(5-2\sqrt{6}\right)^{k+1}\)

\(=\left(5+2\sqrt{6}\right)\left(b-\left(5-2\sqrt{6}\right)^k\right)+\left(5-2\sqrt{6}\right)\left(b-\left(5+2\sqrt{6}\right)^k\right)\)

\(=b\left(5+2\sqrt{6}\right)-\left(5-2\sqrt{6}\right)^{k-1}+b\left(5-2\sqrt{6}\right)-\left(5+2\sqrt{6}\right)^{k-1}\)

\(=10b-a\)

Vậy ta có điều phải chứng minh

6 tháng 10 2017

b/ Đặt \(S_n=\left(5+2\sqrt{6}\right)^n+\left(5-2\sqrt{6}\right)^n=x^n+y^n\)

Ta có: \(\hept{\begin{cases}x^2=10x-1\\y^2=10y-1\end{cases}}\)

\(\Rightarrow S_{n+2}=x^{n+2}+y^{n+2}=10\left(a^{n+1}+b^{n+1}\right)-\left(a^n+b^n\right)=10S_{n+1}-S_n\)

\(\Rightarrow S_{n+2}+S_n=10S_{n+1}⋮10\)

Tương tự cũng có: \(S_{n+4}+S_{n+2}=10S_{n+3}⋮10\) 

\(\Rightarrow S_{n+4}-S_n⋮10\)

Từ đây ta thấy được \(S_{n+4}\equiv S_n\left(mod10\right)\)

Mà \(S_0=2\)

Vậy với mọi n chia hết cho 4 thì số tận cùng của B là 2.

Quay lại bài toán ta thấy \(1004⋮4\) nên M sẽ có chữ số tận cùng là 2.